College Algebra 7th Edition

Published by Brooks Cole
ISBN 10: 1305115546
ISBN 13: 978-1-30511-554-5

Chapter 6, Matrices and Determinants - Section 6.1 - Matrices and Systems of Linear Equations - 6.1 Exercises - Page 501: 49

Answer

Solution set: $\{(-2,1,3)\}$

Work Step by Step

Start with the augmented matrix, row reduce to reduced row echelon form (Gauss-Jordan.) $\left[\begin{array}{llll} 4 & -3 & 1 & -8\\ -2 & 1 & -3 & -4\\ 1 & -1 & 2 & 3 \end{array}\right]\rightarrow\left(\begin{array}{l} R_{1}\leftrightarrow R_{3}.\\ .\\ . \end{array}\right)$ $\rightarrow\left[\begin{array}{llll} 1 & -1 & 2 & 3\\ -2 & 1 & -3 & -4\\ 4 & -3 & 1 & -8 \end{array}\right]\rightarrow\left(\begin{array}{l} .\\ +2R_{1}.\\ -4R_{1}. \end{array}\right)$ $\rightarrow\left[\begin{array}{llll} 1 & -1 & 2 & 3\\ 0 & -1 & 1 & 2\\ 0 & 1 & -7 & -20 \end{array}\right]\rightarrow\left(\begin{array}{l} -R_{2}.\\ \times(-1).\\ +R_{2}. \end{array}\right)$ $\rightarrow\left[\begin{array}{llll} 1 & 0 & 1 & 1\\ 0 & 1 & -1 & -2\\ 0 & 0 & -6 & -18 \end{array}\right]\rightarrow\left(\begin{array}{l} .\\ .\\ \div(-6). \end{array}\right)$ $\rightarrow\left[\begin{array}{llll} 1 & 0 & 1 & 1\\ 0 & 1 & -1 & -2\\ 0 & 0 & 1 & 3 \end{array}\right]\rightarrow\left(\begin{array}{l} -R_{3}.\\ +R_{3}.\\ . \end{array}\right)$ $\rightarrow\left[\begin{array}{llll} 1 & 0 & 0 & -2\\ 0 & 1 & 0 & 1\\ 0 & 0 & 1 & 3 \end{array}\right]$ Solution set: $\{(-2,1,3)\}$
Update this answer!

You can help us out by revising, improving and updating this answer.

Update this answer

After you claim an answer you’ll have 24 hours to send in a draft. An editor will review the submission and either publish your submission or provide feedback.